SIE Exam - Test 2

अब Quizwiz के साथ अपने होमवर्क और परीक्षाओं को एस करें!

A savings account at a bank is guaranted by which of the following entities? A - FDIC B - SIPC C - Federal Reserve D - Department of Treasury

A

UTMA accounts are opened under the tax ID of the: A - Minor B - Donor C - Parent D - Custodian

A

A customer has a cash balance in her account together with long positions in several securities. She has made no securities transactions in the account during the past 18 months. How often must the firm send her an account statement? A - Monthly B - Quarterly C - Semiannually D - Annually

B

A customer is an officer of a company that is involved in some significant changes. All of the following items are examples of corporate affairs that could be considered inside information if the customer shares them with his registered representative except: A - Pending transactions B - Declared stock dividend C - Top management changes D - Imminent financial liquidity problems

B

A decline in the gross domestic product (GDP) must last for at least how many quarters to be considered a recession? A - 1 quarter B - 2 quarters C - 3 quarters D - 4 quarters

B

The primary purposed of a syndicate desk in the context of an equity offering is to: A - Determine the list of selling shareholders B - Build an order book and allocate the stock C - Solicit interest from investors in the stock offering D - Chaperone company management during the road show

B

To offer its shareholders a privilege to obtain its shares at a fixed price, which of the following product does a corporation issue? A - Puts B - Rights C - Futures D - Preferred stock

B

When is interest on Treasure notes paid A - Quarterly B - Semiannually C - Annually D - At maturity

B

Which of the following investment risks is the greatest risk in a variable life insurance policy? A - Credit risk B - Market risk C - Inflation risk D - Interest rate risk

B

A 42‐year‐old investor wants to put $20,000 into a plan to help meet the educational expenses of his 12‐year‐old son. He wants to make a lump sum deposit. Which would you recommend? 529 plan. Coverdell IRA. Roth IRA. Growth mutual fund.

(a) A 529 plan would allow the investor to make a lump sum deposit.

Which of the following is true? Representatives and broker dealers may not disclose any information regarding a client to a third party without the client's expressed consent or a court order. A representative may not obtain outside employment because of the potential conflict of interest. A client may not have a numbered account for his investment account. Broker dealers may not give gifts to the employees of other broker dealers.

(a) A client may have a numbered account if his signature as owner is on file; broker dealers may give gifts to the employees of other broker dealers with certain restrictions. To obtain outside employment, a representative must first obtain approval from the member firm where he works.

As it relates to a member firm conducting business with the public, all of the following are violations, except: Charging a customer a larger than normal commission for executing a specific order. Failing to execute a customer's order for a speculative security. Stating that a new issue has been approved for sale by the SEC. Printing "FINRA" in large type on business cards.

(a) A member firm may charge a customer a larger than ordinary commission for the execution of a specific order so long as it is disclosed to the customer. A member firm must always execute a customer's order.

A testimonial by a compensated expert, citing the results she realized following a member's recommendations, must include which of the following? A statement detailing the expert's credentials. A statement that past performance is not a guarantee of future performance. A statement that the individual is a compensated spokesperson. The name of the principal who approved the ad. I, II, and III. II and IV. I and II. I, II, III, and IV.

(a) All of the choices listed must be included, except for the name of the principal who approved the ad for use.

Before opening a new account for any customer, a registered representative must: Fill out and submit the account for principal review. Send a declaration of investor intent (DII) to the IRS. Fill out a new account form and present it to the investor for his signature. Fill out and sign a full financial declaration.

(a) Before opening a new account for any customer, a registered representative must fill out and sign a new account form, which does not require the signature of the customer.

An investor who may lose part or all of his investment is subject to which of the following? Capital risk. Market risk. Reinvestment risk. Credit risk.

(a) If an investor may lose part or all of his capital, it is called capital risk.

In which type of account does the nominal owner of the account enter all orders for the beneficial owner of the account? Custodial account. Fiduciary account. Authorized account. Discretionary account.

(a) In a custodial account, the custodian is the nominal owner of the account and carries on all transactions for the minor, the real beneficial owner of the account.

A customer and his spouse have an account registered as joint tenants in common. If the customer dies, what would happen to the account? The decedent's assets will be distributed according to his will. The executor of the estate will determine how all of the assets are to be distributed. All of the assets in the account will be distributed according to the trustee. The spouse would get the assets in the account.

(a) The assets of the decedent will be distributed according to the decedent's will.

35. Which of the following is the minimum denomination of Treasury notes that can be purchased by an investor? A. $50 B. $100 C. $1,000 D. $10,000

35. B Treasury notes are sold in minimum denominations of $100.

A customer has a large position in GJH, a thinly traded stock whose share price has remained flat for some time. The customer contacts the agent and wants to sell his entire position. The customer is most subject to which of the following? Liquidity risk. Credit risk. Conversion risk. Execution risk.

(a) The investor has a large position in a thinly traded stock; as a result, the investor is subject to a large amount of liquidity risk.

A client who is 65 years old has invested $10,000 in a Roth IRA. It has now grown to $14,000. He plans to retire and take a lump sum distribution. He will pay taxes on: $0. $14,000. $4,000. $10,000.

(a) The money has been deposited in a Roth IRA after taxes. It is allowed to grow tax deferred. If you are over 59.5 and the money has been in the IRA for at least 5 years, then it may all be withdrawn without paying taxes on the growth.

The nominal owner of an UGMA account is the: Custodian. Minor. Trustee. Parent.

(a) The nominal owner of a UGMA account is the custodian.

You are the owner of a restaurant and you would like to have a guitarist play in the lounge on Saturday evenings. You have known your representative for 15 years and know her to be a great jazz guitarist. You think she would like to play and ask her if she is available to do so. She would have to notify which of the following before accepting your offer? Her firm. No one, because it is not securities related and she is on her own time. FINRA. NYSE.

(a) The representative would have to notify her employer before working outside the office in any capacity.

Two brothers, both married with children, have opened an account with your firm as JTWROS. One brother has passed away. All of the following will not happen with regard to the account except: All assets will become the property of the surviving party. The account will be retitled in the name of the surviving party. The portion of the assets belonging to the deceased will go to his estate. The original account will become an individual account.

(a) This is a joint account with rights of survivorship. All assets become the property of the surviving party.

A representative may borrow money from a client: If the client is the issuer of securities. If the client is a credit union. If the client is a wealthy individual who regularly makes private loans. Under no circumstances.

(b) A representative may borrow from a client if the client is in the business of making loans (i.e., a bank or credit union).

Which of the following is NOT required in the account title for a custodial account? The state. The minor's social security number. The name of the custodian. UGMA.

(b) Although the minor's social security number is listed on the account, it does not appear in the account title.

An investor who is concerned with changes in interest rates would be least likely to purchase which of the following? Long‐term warrants. Long‐term corporate bonds. Long‐term equity. Call options.

(b) An investor who is concerned with changes in interest rates would be least likely to purchase long‐term bonds. As interest rates change, the price of the long‐term bonds will fluctuate the most.

An investor is looking for a risk‐free investment. An agent should recommend which of the following to this investor? Preferred stock. 90‐day T‐bill. Convertible preferred stock. Bankers' acceptances.

(b) Bankers' acceptances are money market instruments and are short term; series HH government bonds can only be exchanged for mature series EE; and convertible preferred stock is a security with risk. A 90‐day T‐bill is considered a risk‐free investment.

Your brokerage firm has placed an ad in the local newspaper, advertising its new line of services being offered to investors. The firm must maintain the ad for how long? 24 months. 36 months. 12 months. 18 months.

(b) Brokerage firms must maintain their advertising for at least 3 years.

In which type of account does a trustee enter all orders for the owners of the account? Custodial account. Fiduciary account. Authorized account. Discretionary account.

(b) In a fiduciary account, the trustee enters all orders for the owners of the account.

An investor has a conservative attitude towards investing and is seeking to invest $50,000 into an interest‐bearing instrument that will provide current income and safety. You would most likely recommend which of the following? Treasury bill. Ginnie Mae pass‐through certificate. Treasury STRIP. Bankers' acceptance.

(b) Of all the investments listed, only the Ginnie Mae pass‐through certificate will provide income. Ginnie Mae pass‐through certificates pay monthly interest and principal payments.

At a member firm, which of the following must be registered? A corporate officer whose sole function is to act as liaison between the board of directors and management. A part‐time sales assistant who occasionally takes verbal orders from customers. A back‐office margin clerk who assists the head of the margin department. A receptionist who takes messages from customers inquiring about their accounts.

(b) The part‐time sales assistant who takes orders from customers must be registered because she is taking orders.

A firm has been taken to arbitration by a customer. The disputed amount is $47,400. Which of the following is true? There will be a hearing, and the decision may be appealed. There will not be a hearing, and the decision may not be appealed. There will be a hearing, and the arbitrator's decision is final. There will be a hearing with up to three arbitrators.

(b) There will be no hearing unless specifically requested by a public customer, and the decision of the arbitrator is final and binding. Claims under $50,000 will be resolved in simplified arbitration.

An investor gets advance notice of a research report being issued and enters an order to purchase the security that is the subject of the research report. This is known as: Front running. Trading ahead. Insider trading. Advance trading.

(b) This is a violation known as trading ahead.

Creating false activity in a security to attract new purchases is a fraudulent practice known as: Trading ahead. Painting the tape. Active concealment. Front running.

(b) This is known as painting the tape, matched purchases, or matched sales.

A couple in their early thirties are seeking an investment for the $40,000 they have saved. They are planning on purchasing a new home in the next 2 years. You should most likely recommend which of the following? Preferred stock. Common stock and common stock funds. Money market funds. Municipal bonds.

(c) A money market fund is the best recommendation for investors who will need access to their funds in the next few years.

A registered representative may accept orders for a client's account from which of the following? Client. Client's spouse. Client's attorney. Client's investment adviser. I and II. I, II, and III. I only. I, II, III, and IV.

(c) A registered representative may only accept orders from the client.

FINRA considers which of the following to be classified as retail communication? Video displays. Listings in phone directories. Circulars. Telemarketing scripts. II and III. I and II. I, II, III, and IV. I and III.

(c) All of the choices listed would be considered retail communication, if any part of the communications listed could be seen by an individual investor.

To open a guardian account, the firm must obtain: Trust papers. Power of attorney. Declaration papers. Affidavit of domicile.

(c) An account set up by a guardian must be accompanied by declaration papers

In the securities industry, which of the following is the ultimate industry authority regulating conduct? NYSE. SRO. SEC. FINRA.

(c) The SEC is the ultimate industry authority in regulating conduct.

You have just opened up a new account for a customer. You are required to have all of the following, except the: Agent's name or ID number. Principal's signature. Customer's signature. Customer's social security number.

(c) The customer is not required to sign anything when opening a new account.

A potential customer that you have been trying to get to open an account with you for some time has agreed to put some money in a mutual fund you have recommended. Which of the following customer information is NOT required on the new account form? Address. Social security number. Educational information. Investment objective.

(c) The customer's educational information is not required on the new account form.

An investor has deposited $100,000 into a qualified retirement account over a 10‐year period. The value of the account has grown to $175,000 and the investor plans to retire and take a lump sum withdrawal. The investor will pay: Capital gains tax on $75,000 only. Ordinary income taxes on the $75,000 only. Ordinary income taxes on the whole $175,000. Ordinary income taxes on the $100,000 and capital gains on the $75,000.

(c) The retirement account is qualified, which means the investor has deposited the money pretax, therefore, all of the money is taxed when it is withdrawn.

A school principal has deposited $15,000 in a tax‐deferred annuity through a payroll deduction plan. The account has grown in value to $22,000. The principal plans to retire and take a lump sum distribution. On what amount does he pay taxes? $22,000. $15,000. $7,000. $0.

(c) This is a nonqualified plan, meaning the money is deposited after taxes so the retiree will only pay taxes on the growth.

Mr. and Mrs. Jones, a couple in their early forties, enjoy watching their son play baseball on the weekends. He is planning to go to college 11 years from September and they are looking to start saving for college expenses. Which of the following would you recommend? Educational IRA. Growth fund. Treasury STRIPS. Custodial account.

(c) Treasury STRIPS are government‐issued zero‐coupon bonds. They are issued at a discount and mature at par, or $1,000. For the exam, they are the best answer for college expense planning if the question is asking for an investment recommendation, not the type of account that it is deposited into.

Which of the following is an associated person of a member firm? Registered representative. Trader. Director. Manager. I and III. I and II. I, II, and IV. I, II, III, and IV.

(d) All of the choices listed are associated people.

Which of the following is NOT allowed as a joint account? A registered representative and a customer. A registered representative and a spouse. A registered representative and a friend. A registered representative and his 16‐year‐old child.

(d) An adult may never have a joint account with a minor

Which of the following could be subject to an investor's AMT? A limited partnership. An open‐end mutual fund. A convertible preferred stock owned by a wealthy investor. An industrial revenue bond.

(d) An industrial revenue bond may subject some wealthy investors to the alternative minimum tax.

A new investor is in the 15% tax bracket and is seeking some additional current income. Which of the following would you recommend? Growth fund. Government bond fund. Municipal bond fund. Corporate bond fund.

(d) An investor in a low tax bracket seeking current income would be best suited for a corporate bond fund.

An investor has a conservative attitude toward investing and is seeking to invest $100,000 into an instrument that will provide current income and the most protection from interest rate risk. You would most likely recommend which of the following? Ginnie Mae pass‐through certificate. Bankers' acceptance. Treasury STRIP. A portfolio of T‐bills.

(d) An investor seeking protection from interest rate risk will most likely be best suited for a portfolio of Treasury bills. As the bills mature, the investor can roll over the position into newly issued bills with new interest rates.

According to Rule 135, as it relates to generic advertising, which of the following is NOT true? The ad may contain information about the services a company offers. The ad may describe the nature of the investment company's business. The ad may contain information about exchange privileges. The ad may contain information about the performance of past recommendations.

(d) Generic advertising may not contain information about past recommendations.

Which of the following is not a violation of the rules of conduct? Recommending a security because of its future price appreciation. Recommending a mutual fund based on a pending dividend to an investor seeking income. Implying that FINRA has approved the firm. Showing a client the past performance of a mutual fund for the last 3 years since its inception.

(d) Showing a client the past performance for a mutual fund that has only been around for 3 years is in line with the regulations. All of the other choices are violations.

Which act gave the NASD (now part of FINRA) the authority to regulate the OTC market? The NASD Act of 1929. The Securities Act of 1933. The Securities Act of 1934. The Maloney Act of 1938.

(d) The Maloney Act of 1938 was an amendment to the Securities Exchange Act of 1934 and established the NASD (now part of FINRA) as the self‐regulatory organization for the over‐the‐counter market.

The Securities Exchange Act of 1934 regulates which of the following markets? Third. Fourth. Primary. Secondary.

(d) The Securities Exchange Act of 1934 regulates the secondary market.

A FINRA member has failed to receive a stock certificate in good form from the selling FINRA firm. Which FINRA bylaw defines good delivery? Rules of Fair Practice. Code of Procedure. Code of Arbitration. Uniform Practice Code.

(d) The Uniform Practice Code regulates the way members conduct business with other members.

A self‐employed individual may open a SEP IRA to plan for his retirement. The maximum contribution to the plan is: $4,000. $8,000. $16,000. The lesser of 25% of the post‐contribution income, up to $55,000.

(d) The maximum contribution for a SEP IRA is the lesser of 25% of the post contribution income or $55,000.

A customer calls in asking about how to put money aside for his children. He wants to open a custodial account for his two children, Bobby and Sue. What should you recommend? Open two accounts for both children, with him and his wife as custodian. Open two accounts for the two children, with him being the custodian on one and his wife being custodian on the other, as one parent may only be custodian for one child. Open one account immediately for both children. Open two accounts, one for each child, with he or his wife as custodians for both or for either.

(d) The rule is one custodian and one minor for each UGMA account. There is not a rule regarding who must be custodian.

The maximum allowable gift to a minor under UGMA is: $15,000. $1,500. $30,000. There is no limit.

(d) There is no limit to the size of the gift that may be given to anyone, but $15,000 per year is the tax‐free limit.

A client has phoned in concerned about what will happen to his investment in a waste management company if the new EPA laws are enacted, requiring disposal companies to reduce pollution. What type of risk is he concerned with? Call risk. Environmental risk. Investment risk. Legislative risk.

(d) This client is concerned about legislative risk, which is the risk that the government will do something that adversely affects an investment.

Which of the following is true? If an investor buys shares just prior to the ex date, he will have his investment money returned. After an investor's money is returned, the investor is still liable for taxes on the dividend amount. A registered representative may not use the pending dividend payment as the sole basis for recommending a stock purchase. All of the above.

(d) Using the pending dividend to create an urgency on the part of the investor to purchase this stock is a perfect example of this violation, and the results are listed in the first three answers.

1. All but which of the following statements are correct regarding bonds and preferred stock? A. If a company declares bankruptcy, bondholders are repaid before preferred stock shareholders. B. Preferred stocks pay dividends; bonds pay interest. C. Bonds are subject to greater interest rate risk than preferred stock. D. Neither bond interest nor preferred stock dividends qualify for capital gains treatment.

1. C Preferred stock does not have a fixed maturity date. Therefore, it has unlimited interest rate risk. If a company declares bankruptcy, bondholders are repaid before preferred stock shareholders. Preferred stocks pay dividends; bonds pay interest. Neither bond interest nor preferred stock dividends qualify for capital gains treatment.

1. How frequently will a mutual fund with a turnover ratio of 20% replace its total holdings? A. Every year B. Every 2 years C. Every 4 years D. Every 5 years

1. D 1 ÷ 0.20 = 5 A mutual fund with a turnover ratio of 20% will replace its total holdings every 5 years.

10. Pete, an elderly business owner, is interested in establishing a retirement plan that will provide the greatest retirement benefit to himself. Which of the following plans should Pete select? A. Defined benefit plan B. Defined contribution plan C. Money purchase plan D. 401(k) plan

10. A A defined benefit plan favors older owner/employees and would provide the greatest retirement benefit to Pete.

10. A health savings account (HSA) allows the account holder to set aside money on a/an __________ basis to pay for __________ medical expenses. A. after-tax, qualified B. after-tax, unqualified C. pre-tax, qualified D. pre-tax, unqualified

10. C A health savings account (HSA) allows the account holder to set aside money on a pre-tax basis to pay for qualified medical expenses.

11. Which of the following is the central banking system of the United States? A. Federal Reserve B. FOMC C. U.S. Mint D. U.S. Treasury

11. A The Federal Reserve is the central banking system of the United States.

11. The S&P index has __________ risk. A. non-systematic B. non-diversifiable C. diversifiable D. unsystematic

11. B The S&P index has systematic risk only. Systematic risk is also referred to as non-diversifiable risk. The other answers (non-systematic risk, diversifiable risk, and unsystematic risk) all refer to the same type of risk.

12. If an investor's brokerage firm goes out of business and is a member of the SIPC, then the investor's cash and securities held by the firm may be protected up to __________, including a __________ limit for cash. A. $500,000, $250,000 B. $500,000, $500,000 C. $1,000,000, $250,000 D. $1,000,000, $500,000

12. A If an investor's brokerage firm goes out of business and is a member of the SIPC, then the investor's cash and securities held by the firm may be protected up to $500,000, including a $250,000 limit for cash.

12. Which of the following mutual fund share classes will charge investors a back-end load? A. Class A shares B. Class B shares C. Class C shares D. Class D shares

12. B Class B mutual fund shares charge a back-end load.

13. All but which of the following are characteristics of defined contribution plans? A. Employer contributions are defined. B. The employee assumes the risk of investment performance. C. The employer assumes the risk of pre-retirement inflation. D. Benefits cannot be provided for past service.

13. C In a defined contribution plan, employer contributions are defined, and benefits cannot be provided for past service. The employee assumes the risk of investment performance and pre-retirement inflation.

13. Regulation A permits unregistered public offerings of up to ______ of securities in any 12-month period. A. $1M B. $2M C. $5M D. $10M

13. C Regulation A permits unregistered public offerings of up to $5 million of securities in any 12-month period.

14. Which of the following is a requirement in which financial institutions need to verify the identity of individuals wishing to conduct financial transactions with them? A. Automated Client Account Transfer Service B. Customer Identification Program C. Trade Reporting and Compliance Engine D. None of the above are correct.

14. B The Customer Identification Program (CIP) is a requirement in which financial institutions need to verify the identity of individuals wishing to conduct financial transactions with them.

14. A "penny stock" is an equity security that trades below: A. $0.05. B. $0.50. C. $1.00. D. $5.00.

14. D A "penny stock" is an equity security that trades below $5.

15. Which of the following forms of ownership allows an investor to own securities without a certificate? A. Book entry form B. Central entry form C. Custodian entry form D. Depository entry form

15. A The book entry form of ownership allows an investor to own securities without a certificate.

15. The FDIC maintains stability and public confidence in the nation's financial system by doing all but which of the following? A. Insuring deposits. B. Managing receiverships. C. Examining and supervising financial institutions for safety, soundness, and consumer protection. D. Setting minimum standards for voluntarily established pension and health plans.

15. D The FDIC maintains stability and public confidence in the nation's financial system by insuring deposits, managing receiverships, and examining and supervising financial institutions for safety, soundness, and consumer protection. ERISA, not the FDIC, sets minimum standards for voluntarily established pension and health plans.

16. Which of the following provides clearing, settlement, risk management, central counterparty services, and a guarantee of completion for certain transactions for virtually all broker-to-broker trades involving equities, corporate and municipal debt, ADRs, ETFs, and UITs? A. ATC B. NSCC C. OCC D. TRACE

16. B The NSCC (National Securities Clearing Corporation) provides clearing, settlement, risk management, central counterparty services, and a guarantee of completion for certain transactions for virtually all broker-to-broker trades involving equities, corporate and municipal debt, ADRs, ETFs, and UITs.

16. Which of the following has the mission of protecting investors, municipal entities, and the public interest by promoting a fair and efficient municipal market, regulating firms that engage in municipal securities and advisory activities, and promoting market transparency? A. AMBAC B. FNMA C. GNMA D. MSRB

16. D The MSRB (Municipal Securities Rulemaking Board) has the mission of protecting investors, municipal entities, and the public interest by promoting a fair and efficient municipal market, regulating firms that engage in municipal securities and advisory activities, and promoting market transparency.

21. Which of the following investments will provide tax-exempt interest if the proceeds are used to pay for qualifying education expenses? (1) FNMA funds (2) Series EE bonds (3) Treasury bonds (4) Treasury bills A. (2) only B. (1) and (2) only C. (2), (3), and (4) only D. All of the above are correct.

21. A Series EE bonds provide tax-exempt interest if the proceeds are used to pay for qualifying education expenses.

For questions 17 - 19, match the real estate investment with the description that follows. Use only one answer per blank. Answers may be used more than once or not at all. A. Equity REIT B. Mortgage REIT C. REMIC 17. ____ Invests in loans secured by real estate. 18. ____ Self-liquidating, flow-through entity that invests in real estate mortgages or mortgage-backed securities. 19. ____ Acquires ownership interests in commercial, industrial, and residential properties. Income is received from the rental of these properties.

17. B Mortgage REITs invest in loans secured by real estate. 18. C A REMIC is a self-liquidating, flow-through entity that invests in real estate mortgages or mortgage-backed securities. 19. A Equity REITs acquire ownership interests in commercial, industrial, and residential properties. Income is received from the rental of these properties.

17. Which of the following must be filed for each of the first three fiscal quarters of a company's fiscal year, and includes unaudited financial statements and provides a continuing view of the company's financial position during the year? A. Form ADV B. Form 8-K C. Form 10-K D. Form 10-Q

17. D Form 10-Q must be filed for each of the first three fiscal quarters of a company's fiscal year. It includes unaudited financial statements and provides a continuing view of the company's financial position during the year.

18. According to SEC Rule 506(b), private placements may be sold to how many accredited and non-accredited investors? A. An unlimited number of accredited investors and up to 35 non-accredited investors. B. An unlimited number of accredited investors and up to 100 non-accredited investors. C. A total of 35 accredited and non-accredited investors. D. A total of 100 investors, of which 35 can be non-accredited.

18. A According to SEC Rule 506(b), private placements may be sold to an unlimited number of accredited investors and up to 35 non-accredited investors.

19. Kappa Inc., a C Corp, had a profitable year and has extra money to invest. The owners would like to maximize the after-tax income to the corporation. Which of the following investments would best help them achieve their goal? A. Value stocks B. Preferred stock C. Municipal bonds D. Highly-rated corporate bonds

19. B Corporate investors in preferred stock can generally deduct 70% of the dividends they receive.

2. Which of the following is correct regarding a pre-emptive right? A. It is also referred to as a "subscription right" or a "subscription privilege." B. It is an obligation for existing shareholders to purchase new shares of stock before they are offered to the public. C. Exercising a pre-emptive right can cause dilution. D. All of the above are correct.

2. A A pre-emptive right is also referred to as a "subscription right" or a "subscription privilege." It is a right, but not an obligation for existing shareholders to exercise, and it prevents dilution among existing shareholders.

2. All but which of the following are considered securities under the Uniform Securities Act? A. Debentures B. Precious metals C. Variable annuities D. Variable life insurance

2. B Debentures, variable annuities, and variable life insurance are considered securities under the Uniform Securities Act. Precious metals are not considered securities.

20. Which of the following regulations contains rules providing exemptions from the registration requirements under the Securities Act of 1933? A. Regulation A B. Regulation G C. Regulation T D. Regulation U

20. A Regulation A contains rules providing exemptions from the registration requirements under the Securities Act of 1933.

20. According to FINRA Rule 3240, which of the following is correct regarding borrowing and lending arrangements between a Registered Investment Advisor and its customers? A. The specific borrowing/lending arrangement must meet certain conditions, such as the customer cannot be an immediate family member. B. Proper notification of the borrowing or lending arrangement must be given, but advanced approval is not required. C. The member firm must have a written policy in place regarding borrowing and lending arrangements. D. All of the above are correct.

20. C According to FINRA Rule 3240, borrowing and lending arrangements between a Registered Investment Advisor and its customers requires the member firm to have a written policy in place.

21. Which of the following describes the maturities of Treasury bills, Treasury notes, and Treasury bonds? A. Treasury notes have maturities of 10 years or more. B. Treasury bills have maturities of 1 year or more. C. Treasury bonds have maturities greater than 10 years. D. All of the above are correct.

21. C Treasury bills have maturities of 1 year or less. Treasury notes have maturities of 10 years or less. Treasury bonds have maturities greater than 10 years.

22. Which of the following type of bond is unregistered, with no record kept regarding the owner or transactions involving ownership? A. Bearer bond B. Brady bond C. Fidelity bond D. Surety bond

22. A A bearer bond is a type of bond that is unregistered, with no record kept regarding the owner or transactions involving ownership.

22. Which of the following is a set of documents, including a prospectus, which a company must file with the SEC pursuant to the Securities Act of 1933 before it proceeds with a public offering? A. Prospectus statement B. Red herring C. Registration statement D. Tombstone ad

22. C A registration statement is a set of documents, including a prospectus, which a company must file with the SEC pursuant to the Securities Act of 1933 before it proceeds with a public offering.

23. If the yield curve is __________, then the spread between yields of short-term and long-term bonds is __________. A. flattening, decreasing B. flattening, increasing C. steepening, decreasing D. steepening, increasing

23. A If the yield curve is flattening, then the spread between yields of short-term and long-term bonds is decreasing.

23. Which of the following is/are correct regarding liquidity and marketability? (1) Liquidity is the ability to sell or redeem an investment quickly and at a known price without incurring a significant loss of principal. (2) Marketability is the speed and ease with which an investment may be bought or sold. A. (1) only B. (2) only C. Both (1) and (2) are correct. D. Neither (1) or (2) are correct.

23. C Liquidity is the ability to sell or redeem an investment quickly and at a known price without incurring a significant loss of principal. Marketability is the speed and ease with which an investment may be bought or sold.

For questions 24 - 27, match the form of dividend with the description that follows. Use only one answer per blank. Answers may be used more than once or not at all. A. Stock dividend B. Credit dividend C. Ordinary dividend D. Qualified dividend E. Constructive dividend F. Liquidating dividend G. Tax dividend 24. ____ A distribution made by a corporation that is paid as additional shares of stock rather than cash. 25. ____ A type of dividend to which capital gains tax rates are applied. 26. ____ Normally a disguised dividend, such as a below-market shareholder loan. 27. ____ A payment to shareholders that exceeds the company's retained earnings; payment is made from capital rather than earnings.

24. A Stock dividends are distributions by a corporation that are paid as 25. D A qualified dividend is a type of dividend to which capital gains tax rates are applied. 26. E A disguised dividend, such as a below-market shareholder loan, is considered to be a constructive dividend. 27. F A payment to shareholders that exceeds the company's retained earnings is a liquidating dividend. The payment is made from capital rather than earnings.

24. Which of the following are among the criteria to be an "accredited investor"? A. Net worth exceeding $500,000, either alone or together with a spouse, excluding the value of the person's primary residence. B. Net worth exceeding $500,000, either alone or together with a spouse, including the value of the person's primary residence. C. Net worth exceeding $1,000,000, either alone or together with a spouse, excluding the value of the person's primary residence. D. Net worth exceeding $1,000,000, either alone or together with a spouse, including the value of the person's primary residence.

24. C To be considered an accredited investor, net worth must exceed $1,000,000, either alone or together with a spouse, excluding the value of the person's primary residence.

25. All but which of the following are characteristics of American Depository Receipts (ADRs)? A. They are traded on secondary exchanges. B. They represent ownership interest in foreign securities denominated in U.S. dollars. C. They involve banks collecting money in U.S. dollars and then converting into foreign currency for ADR holders. D. They are issued by banks in foreign countries.

25. C For ADRs, banks collect money in their local currency and then convert to U.S. dollars. ADRs are traded on secondary exchanges and represent ownership interest in foreign securities denominated in U.S. dollars. They are issued by banks in foreign countries.

26. Which of the following accurately describes the difference between rights and warrants? (1) A warrant may be attached to new debt or preferred issues to make the issues more attractive to buyers. (2) Rights and warrants have different lifespans. (3) Warrants usually expire within a few weeks. (4) Rights may continue without expiring for up to several years. A. (1) only B. (1) and (2) only C. (2), (3), and (4) only D. All of the above are correct.

26. B A warrant may be attached to new debt or preferred issues to make the issues more attractive to buyers. A difference between rights and warrants is their lifespan. Rights usually expire within a few weeks, and warrants may continue without expiring for up to several years.

27. Which of the following is the governing body of FINRA, which oversees the administration of its affairs and the promotion of its welfare, objectives, and purposes? A. Board of Governors B. Board of Trustees C. Congress D. FINRA Chairman

27. A The Board of Governors is the governing body of FINRA and oversees the administration of its affairs and the promotion of its welfare, objectives, and purposes.

28. Which of the following are backed by the full faith and credit of the government issuing the bonds and are repaid through taxes collected by the government body? A. General obligation bonds B. Moral obligation bonds C. Private purpose bonds D. Revenue bonds

28. A General obligation bonds are backed by the full faith and credit of the government issuing the bonds and are repaid through taxes collected by the government body.

28. Preferred stock that does not have to pay missed dividends is considered which of the following? A. Convertible B. Cumulative C. In-kind D. Noncumulative

28. D Preferred stock that does not have to pay missed dividends is considered noncumulative.

34. The two main types of mutual fund prospectuses are the __________ prospectus and the __________ prospectus. A. omitting, statutory B. preliminary, statutory C. preliminary, summary D. statutory, summary

34. D The two main types of mutual fund prospectuses are the statutory prospectus and the summary prospectus.

29. Which of the following rules permits the public resale of restricted or control securities if a number of conditions are met, including how long the securities are held, the way in which they are sold, and the amount that can be sold at any one time? A. Securities Act Rule 144 B. Securities Act Rule 405 C. Securities Act Rule 433 D. Securities Act Rule 506

29. A Securities Act Rule 144 permits the public resale of restricted or control securities if a number of conditions are met, including how long the securities are held, the way in which they are sold, and the amount that can be sold at any one time.

29. All but which of the following are correct regarding Electronic Communication Networks (ECNs)? A. They are a type of alternative trading system (ATS). B. They trade unlisted stocks and other non-exchange-traded products. C. Unlike dark pools, ECNs display orders in the consolidated quote stream. D. ECNs are required to register with the SEC as broker-dealers.

29. B Electronic Communication Networks (ECNs) are a type of alternative trading system (ATS) that trades listed stocks and other exchange-traded products. Unlike dark pools, ECNs display orders in the consolidated quote stream. ECNs are required to register with the SEC as broker-dealers.

3. Which of the following is a characteristic of American Depository Receipts (ADRs)? A. ADRs trade once per day like mutual funds. B. ADR holders receive foreign tax credits for income tax paid to a foreign country. C. ADR dividends are declared in U.S. dollars. D. ADRs allow domestic securities to be traded in foreign countries.

3. B ADRs allow for the trading of international securities in domestic countries. They trade throughout the day, and their dividends are declared in local currencies and paid in U.S. dollars. ADR holders receive foreign tax credits for income tax paid to a foreign country.

3. When a bond is selling at a premium to par, the yield to maturity (YTM) will always be __________ the bond's coupon rate. If a bond is selling at a discount to par, the YTM will always be __________ the bond's coupon rate. A. greater than, equal to B. greater than, less than C. less than, equal to D. less than, greater than

3. D When a bond is selling at a premium to par, the yield to maturity (YTM) will always be less than the bond's coupon rate. If a bond is selling at a discount to par, the YTM will always be greater than the bond's coupon rate.

30. Which of the following is/are correct regarding a flat yield curve? (1) It generally indicates an economic slowdown. (2) It occurs when there is little difference between short-term and long-term yields for debt instruments of the same credit quality. A. (1) only B. (2) only C. Both (1) and (2) are correct. D. Neither (1) or (2) are correct.

30. C A flat yield curve generally indicates an economic slowdown. It occurs when there is little difference between short-term and long-term yields for debt instruments of the same credit quality.

30. Which of the following was established by FINRA as an integrated audit trail of order, quote, and trade information for all National Market System stocks and OTC equity securities? A. ACATS B. EMMA C. NBBO D. OATS

30. D OATS (Order Audit Trail System) was established by FINRA as an integrated audit trail of order, quote, and trade information for all National Market System stocks and OTC equity securities.

31. Which of the following is correct regarding the selling price of Treasury STRIPS? A. They are sold at a discount from face value. B. They are sold at a premium to face value. C. They are sold at face value. D. They are sold at either a discount or premium to face value.

31. A Treasury STRIPS are sold at a discount from face value.

31. Which of the following SEC regulations requires firms to have policies and procedures addressing the protection of customer information and records? This regulation also requires firms to provide initial and annual privacy notices to customers describing information sharing policies. A. Regulation FD B. Regulation MA-W C. Regulation N-Q D. Regulation S-P

31. D Regulation S-P requires firms to have policies and procedures addressing the protection of customer information and records. This regulation also requires firms to provide initial and annual privacy notices to customers describing information sharing policies.

32. According to the Telephone Consumer Protection Act, solicitors are prohibited from calling residences before __________ and after __________ local time. A. 7 a.m., 10 p.m. B. 8 a.m., 9 p.m. C. 9 a.m., 5 p.m. D. 10 a.m., 8 p.m.

32. B According to the Telephone Consumer Protection Act, solicitors are prohibited from calling residences before 8 a.m. and after 9 p.m. local time.

32. All but which of the following are correct regarding the IARD? A. It was developed according to the requirements of its sponsors, the SEC and NASAA. B. Its database helps promote uniformity through the use of common forms, and efficiency through a paperless environment. C. It is to investment advisers what the CRD is to broker-dealers. D. All of the above are correct.

32. D The IARD was developed according to the requirements of its sponsors, the SEC and NASAA. Its database helps promote uniformity through the use of common forms, and efficiency through a paperless environment. It is to investment advisers what the CRD is to broker-dealers.

33. Which of the following describes the illegal practice of excessive buying and selling of securities in a customer's account without considering the customer's investment goals? Its primary goal is to generate commissions that benefit the broker. A. Capping B. Churning C. Front running D. Painting the tape

33. B Churning describes the illegal practice of excessive buying and selling of securities in a customer's account without considering the customer's investment goals. Its primary goal is to generate commissions that benefit the broker.Churning describes the illegal practice of excessive buying and selling of securities in a customer's account without considering the customer's investment goals. Its primary goal is to generate commissions that benefit the broker.

36. Treasury STRIPS are always issued at a: A. premium to par. B. discount to par. C. price equal to par. D. price above or equal to par.

36. B Treasury STRIPS are always issued at a discount to par, like zero-coupon bonds.

37. Which of the following is used to determine whether an instrument qualifies as an "investment contract" for the purposes of the Securities Act of 1933? A. The Howey test B. The Knight test C. The Lochner test D. The Ralston test

37. A The Howey test is the result of the U.S. Supreme Court case, Securities and Exchange Commission v. W. J. Howey Co. from 1946. It is used to determine whether an instrument qualifies as an "investment contract" for the purposes of the Securities Act of 1933.

38. Which of the following securities are backed by the full faith and credit of the U.S. government? A. Federal Home Loan Mortgage Corporation debentures (Freddie Macs) B. Federal National Mortgage Association certificates (Fannie Maes) C. Government National Mortgage Association certificates (Ginnie Maes) D. Student Loan Marketing Association notes (Sallie Maes)

38. C Only Government National Mortgage Association certificates (Ginnie Maes) are backed by the full faith and credit of the U.S. government.

39. Which of the following refers to the high-speed electronic system that reports the latest price and volume data on sales of exchange-listed stocks? A. Consolidated tape B. Instinet C. EDGAR D. TRACE

39. A Consolidated tape refers to the high-speed electronic system that reports the latest price and volume data on sales of exchange-listed stocks.

4. Which of the following is the measure of a company's earnings per share if all convertible securities were exercised and converted to common stock? A. Converted earnings per share B. Conversion ratio per share C. Diluted earnings per share D. Price/earnings per share

4. C Diluted earnings per share is the measure of a company's earnings per share if all convertible securities were exercised and converted to common stock.

4. Which of the following elements of FINRA's Continuing Education Program focuses on compliance, ethical, and sales practice standards? Its content is derived from industry rules and regulations, and accepted standards and practices in the industry. A. Compliance Element B. Firm Element C. Regulatory Element D. Regulatory and Firm Elements

4. C The Regulatory Element of FINRA's Continuing Education Program focuses on compliance, ethical, and sales practice standards. Its content is derived from industry rules and regulations, and accepted standards and practices in the industry.

40. All but which of the following are correct regarding a bond's call provision? A. It protects the issuer from declines in interest rates. B. It will cause the investor's required rate of return to be lower. C. It may be included in a bond agreement. D. It allows the debtor to pay off the debt after a specific period of time at a predetermined price.

40. B A bond's call provision may be included in a bond agreement, and it allows the debtor to pay off the debt after a specific period of time at a predetermined price. It protects the issuer from declines in interest rates. If a bond is callable it will cause an investor's required rate of return to be higher.

41. In order to be eligible to make a traditional IRA contribution, an individual must be younger than age __________ by the end of the taxable year. A. 59 ½ B. 65 C. 70 ½ D. 71

41. C In order to be eligible to make a traditional IRA contribution, an individual must be younger than age 70 ½ by the end of the taxable year.

42. All but which of the following are correct regarding the Federal Open Market Committee (FOMC)? A. It consists of 12 members. B. It holds 8 regularly scheduled meetings per year. C. It reviews economic and financial conditions and determines the appropriate stance of monetary policy. D. All of the above are correct.

42. D The Federal Open Market Committee (FOMC) consists of 12 members and holds 8 regularly scheduled meetings per year. It reviews economic and financial conditions and determines the appropriate stance of monetary policy.

43. Which of the following lists the stages of money laundering in the correct order? A. Layering, integration, placement B. Layering, placement, integration C. Placement, integration, layering D. Placement, layering, integration

43. D The stages of money laundering, in the correct order, are placement, layering, integration.

For questions 44 - 47, match the legislation with the description that follows. Use only one answer per blank. Answers may be used more than once or not at all. A. Securities Act of 1933 B. Securities Exchange Act of 1934 C. Investment Company Act of 1940 D. Securities Investor Protection Act of 1970 44. ____ Regulates brokerage firms. 45. ____ Regulates mutual funds. 46. ____ Regulates new securities. 47. ____ Regulates existing securities.

44. D The Securities Investor Protection Act of 1970 regulates brokerage firms. 45. C The Investment Company Act of 1940 regulates mutual funds. 46. A The Securities Act of 1933 regulates new securities. 47. B The Securities Exchange Act of 1934 regulates existing securities.

48. If interest rates __________ following a bond issue, a sinking-fund provision will allow the issuing company to reduce the interest rate risk of its bonds as it replaces a portion of the existing debt with __________ bonds. A. decline, higher yielding B. decline, lower yielding C. rise, higher yielding D. rise, lower yielding

48. B If interest rates decline following a bond issue, a sinking-fund provision will allow an issuing company to reduce the interest rate risk of its bonds as it replaces a portion of the existing debt with lower yielding bonds.

49. Which of the following refers to the inverse relationship between bond prices and an investor's required rate of return? A. Credit risk B. Interest rate risk C. Liquidity risk D. Reinvestment risk

49. B Interest rate risk is the risk that, as interest rates rise, bond prices will fall. Interest rate risk is measured by a bond's duration.

5. Which of the following is the voice of state securities agencies responsible for efficient capital formation and grass-roots investor protection in the U.S.? Their fundamental mission is to protect consumers who purchase securities or investment advice, and their jurisdiction extends to a wide variety of issuers and intermediaries who offer and sell securities to the public. A. NAIC B. NASAA C. NASD D. SIPC

5. B The NASAA (North American Securities Administrators Association) is the voice of state securities agencies responsible for efficient capital formation and grass-roots investor protection in the U.S. Their fundamental mission is to protect consumers who purchase securities or investment advice, and their jurisdiction extends to a wide variety of issuers and intermediaries who offer and sell securities to the public.

5. Which of the following imposed an obligation on the SEC to consider the impacts that any new regulation would have on competition, and empowered the SEC to establish a national market system and a system for nationwide clearing and settlement of securities transactions? A. Uniform Securities Act of 1956 B. Securities Act Amendments of 1975 C. Uniform Prudent Investors Act of 1994 D. National Securities Market Improvement Act of 1996

5. B The Securities Act Amendments of 1975 imposed an obligation on the SEC to consider the impacts that any new regulation would have on competition, and empowered the SEC to establish a national market system and a system for nationwide clearing and settlement of securities transactions.

50. Which of the following describes the process in which an underwriter intervenes in the secondary market by placing a bid for securities at or below the offering price? The goal is to protect the price from dropping if there is a lack of initial interest for a new issue. A. Stabilizing bid B. Standby underwriting C. Subscription underwriting D. Underwriting bid

50. A A stabilizing bid is the process in which an underwriter intervenes in the secondary market by placing a bid for securities at or below the offering price. The goal is to protect the price from dropping if there is a lack of initial interest for a new issue.

51. Which of the following is an internal meeting between the officials of an organization that will be issuing securities and members of the syndicate that will be distributing them? The meeting is held after the registration of a new security with the SEC, but before the registration's effective date. A. Due diligence meeting B. Prospectus meeting C. Registration meeting D. Syndicate meeting

51. A A due diligence meeting is an internal meeting between the officials of an organization that will be issuing securities and members of the syndicate that will be distributing them. The meeting is held after the registration of a new security with the SEC, but before the registration's effective date.

52. The annual report on __________ provides a comprehensive overview of a company's business and financial condition and includes audited financial statements. A. Form 6-K B. Form 8-K C. Form 10-K D. Form 10-Q

52. C The annual report on Form 10-K provides a comprehensive overview of a company's business and financial condition and includes audited financial statements.

53. All but which of the following are types of municipal bonds? A. General obligation bond B. Preferred bond C. Private activity bond D. Revenue bond

53. B Types of municipal bonds include general obligation bonds, private activity bonds, and revenue bonds.

54. Alex, age 70, has contributed $20,000 to a Roth IRA throughout his career. The account value is now $30,000. If Alex withdraws the entire amount, how much tax will he owe if he's in the 28% tax bracket? A. $0 B. $1,500 C. $4,500 D. $8,400

54. A Roth IRA contributions are made with after-tax dollars and grow tax-free. Alex will not owe taxes when he takes the distribution.

55. During a recent recession, your client, Charles, purchased high-yield corporate bonds that now face minimal default risk. However, he's now concerned that the various corporations may decide to call their bonds. You tell Charles that corporations are likely to call their bonds when: A. interest rates are expected to drop. B. the bonds are selling at a significant premium. C. inflation is expected to rise. D. interest rates have declined.

55. B If corporate bonds are selling at a significant premium, then newly issued bonds are selling with lower coupons. The corporations are likely to call their bonds and replace them with lower coupon bonds.

56. Which of the following is an electronic filing system that facilitates investment adviser registration, exempt reporting adviser filing, regulatory review, and the public disclosure information of registered investment adviser firms and individuals? A. ADV B. CRD C. EDGAR D. IARD

56. D The IARD is an electronic filing system that facilitates investment adviser registration, exempt reporting adviser filing, regulatory review, and the public disclosure information of registered investment adviser firms and individuals.

57. Which of the following acts established the policies and procedures commonly referred to as a "Chinese wall"? A. Uniform Securities Act of 1956 B. Bank Secrecy Act of 1970 C. Insider Trading and Securities Fraud Enforcement Act of 1988 D. Sarbanes-Oxley Act of 2002

57. C The Insider Trading and Securities Fraud Enforcement Act of 1988 established the policies and procedures commonly referred to as a "Chinese wall."

58. Which of the following describes the relationship between a security's bid-ask spread and its liquidity? A. In general, the smaller a security's bid-ask spread, the better its liquidity. B. In general, the larger a security's bid-ask spread, the better its liquidity. C. The bid-ask spread effects a security's marketability, but not its liquidity. D. There is no relationship between a security's bid-ask spread and its liquidity.

58. A In general, the smaller a security's bid-ask spread, the better its liquidity.

59. Which of the following is a type of debt issued by a national government in a foreign currency in order to finance the issuing country's growth and development? A. Domestic debt B. Eurodebt C. Foreign debt D. Sovereign debt

59. D Sovereign debt is issued by a national government in a foreign currency in order to finance the issuing country's growth and development.

6. Congress has provided which of the following entities with the power to supervise self-regulatory organizations (SROs) as a matter of public interest? A. FINRA B. MSRB C. NASD D. SEC

6. D Congress has provided the SEC with the power to supervise self-regulatory organizations (SROs) as a matter of public interest.

6. Which of the following is defined as the benefit provided to an asset manager by a broker-dealer as a result of commissions generated from financial transactions executed by the broker-dealer? A. 12b-1 compensation B. Administrative compensation C. Hard-dollar compensation D. Soft-dollar compensation

6. D Soft-dollar compensation is defined as the benefit provided to an asset manager by a broker-dealer as a result of commissions generated from financial transactions executed by the broker-dealer.

60. The two broad categories of defined contribution plans are: A. defined benefit plans and pension plans. B. personal plans and employer plans. C. profit sharing plans and pension plans. D. qualified plans and profit sharing plans.

60. C The two broad categories of defined contribution plans are profit sharing plans and pension plans.

61. The Dow Jones Industrial Average is an index comprised of __________ industrial companies. A. 20 B. 30 C. 40 D. 50

61. B The Dow Jones Industrial Average is an index comprised of 30 industrial companies.

62. Which of the following determines the minimum margin requirement for investment accounts? A. Congress B. FDIC C. Federal Reserve D. SIPC

62. C The Federal Reserve determines the minimum margin requirement for investment accounts.

63. Regarding municipal bonds, which of the following is the small amount of money, usually less than 5% of an issue, that underwriters give to the issuer in exchange for the right to place part of the issue? A. Credit deposit B. Down payment C. Escrow deposit D. Good faith deposit

63. D Regarding municipal bonds, a good faith deposit is the small amount of money, usually less than 5% of an issue, that underwriters give to the issuer in exchange for the right to place part of the issue.

64. All but which of the following are correct regarding a UGMA/UTMA account? A. Unlike a 529 plan, funds do not need to be used to pay for education expenses. B. The custodian, typically the minor's parent, does not own the assets in the account. C. Similar to a 529 plan, funds in the account grow tax-deferred. D. Once the account is set up, it's considered to be an irrevocable gift.

64. C Unlike a 529 plan, funds in a UGMA/UTMA account do not grow tax-deferred. However, the funds in the UGMA/UTMA account do not need to be used to pay for education expenses. The custodian, typically the minor's parent, does not own the assets in the account. Once the account is set up, it's considered to be an irrevocable gift.

65. A currency transaction report (CTR) must be filed by U.S. financial institutions for each deposit, withdrawal, exchange of currency, or other payment to the institution which involves a transaction in currency of more than: A. $5,000. B. $10,000. C. $25,000. D. $50,000.

65. B A currency transaction report (CTR) must be filed by U.S. financial institutions for each deposit, withdrawal, exchange of currency, or other payment to the institution which involves a transaction in currency of more than $10,000.

For questions 66 - 69, match the hedging technique with the description that follows. Use only one answer per blank. Answers may be used more than once or not at all. A. Collar B. Spread C. Straddle D. Protective put 66. ____ Purchasing a call option and selling a call option on the same stock at the same time. 67. ____ Purchasing a put option and selling a call option on the same stock at the same time. 68. ____ Purchasing a put option while holding shares of the underlying stock from a previous purchase. 69. ____ Purchasing a call option and a put option on the same stock at the same time.

66. B A spread is the simultaneous purchase of one option and the sale of another option on the same side or position within the market. For example, purchasing a call option and selling a call option on the same stock at the same time is a spread. 67. A A collar is a technique used to protect an investor's gain in a long position of stock. Specifically, an investor purchases a put option to protect against a decline in the value of an underlying stock, and sells a call option to generate premium income to cover the cost of the put option premium. 68. D In a protective put, an investor purchases a put option while holding 69. C A straddle is the simultaneous purchase of a call option and a put option on the same stock at the same time.

7. Which of the following is/are correct regarding interest paid from municipal bonds? (1) Interest paid from municipal bonds is not taxed by the federal government. (2) Interest paid from municipal bonds is never taxable at the state level. A. (1) only B. (2) only C. Both (1) and (2) are correct. D. Neither (1) or (2) are correct.

7. A Interest paid from municipal bonds is not taxed by the federal government. The bond interest may also be tax-exempt by various states if certain requirements are met.

7. Which of the following is required by the IRS if a taxpayer uses the substantially equal periodic payment (SEPP) exception to the premature distribution penalty? A. The taxpayer must show proof of economic hardship. B. The taxpayer must show proof that payments will be used to pay for qualified medical or education expenses. C. The taxpayer must show proof that appropriate taxes will be withheld. D. None of the above are correct.

7. D For the substantially equal periodic payment (SEPP) exception to the premature distribution penalty, the IRS does not require a reason for taking withdrawals.

70. Which of the following is/are correct regarding a bond's coupon rate? (1) The smaller a bond's coupon, the greater its relative price fluctuation. (2) The smaller a bond's coupon, the greater its reinvestment risk. A. (1) only B. (2) only C. Both (1) and (2) are correct. D. Neither (1) or (2) are correct.

70. A The smaller a bond's coupon, the greater its relative price fluctuation. The smaller a bond's coupon, the lower its reinvestment risk.

71. Which of the following describes the difference between a 401(k) plan and a 403(b) plan? A. A 401(k) plan is a qualified plan, and a 403(b) plan is not a qualified plan. B. A 403(b) plan is a qualified plan, and a 401(k) plan is not a qualified plan. C. A 401(k) plan allows loans, and a 403(b) plan does not allow loans. D. A 501(c)(3) organization may establish a 403(b) plan but cannot establish a 401(k) plan.

71. A A 401(k) plan is a qualified plan, and a 403(b) plan is not a qualified plan.

72. An investor purchased a share of Kappa stock for $184 and sold it for $173. If a $4.25 dividend was paid during the holding period, what was the total return? A. -4.67% B. -4.33% C. -4.02% D. -3.67%

72. D Rt = (Pt - Pt-1 + Dt) ÷ Pt-1 Rt = ($173 - $184 + $4.25) ÷ $184 = -0.0367 = -3.67%

73. A corporate insider is defined as a director or senior officer of a company, as well as any person or entity that beneficially owns more than __________ of a company's voting shares. A. 1% B. 2% C. 5% D. 10%

73. D A corporate insider is defined as a director or senior officer of a company, as well as any person or entity that beneficially owns more than 10% of a company's voting shares.

74. Which of the following is the typical limit on the term of a loan from a qualified retirement plan? A. 1 year B. 2 years C. 5 years D. 10 years

74. C The typical limit on the term of a loan from a qualified retirement plan is 5 years.

75. The Securities Acts Amendments of 1975 did which of the following? A. It gave authority to the Municipal Securities Rulemaking Board (MSRB). B. It required financial institutions to assist U.S. government agencies to detect and prevent money laundering. C. It provided fiduciary responsibilities for those who manage and control plan assets and gave participants the right to sue for benefits and breaches of fiduciary duty. D. It created the Public Company Accounting Oversight Board to oversee the activities of the auditing profession.

75. A The Securities Acts Amendments of 1975 gave authority to the Municipal Securities Rulemaking Board (MSRB).

8. Which of the following is a person who, for compensation, makes recommendations regarding securities, manages client accounts, and determines which advice regarding securities should be given? A. ADV B. CRD C. IAR D. IARD

8. C An IAR (investment adviser representative) is a person who, for compensation, makes recommendations regarding securities, manages client accounts, and determines which advice regarding securities should be given.

8. All but which of the following are correct regarding investment adviser representatives (IARs)? A. Every investment advisory firm must have at least one IAR registered to the firm. B. An IAR can be a dual registrant of multiple firms in some states only. C. There are currently no continuing education requirements for IARs. D. An IAR does not need to be registered to an RIA firm in order to conduct investment advisory business.

8. D In order to conduct investment advisory business, an investment adviser representative (IAR) must be registered to an RIA firm, and every investment advisory firm must have at least one IAR. An IAR can be a dual registrant of multiple firms in some states only. There are currently no continuing education requirements for IARs.

9. Which of the following is an investor protection requirement that obligates a broker-dealer to exercise reasonable care to execute a customer's order in a way to obtain the most advantageous terms for the customer? A. Best execution B. Price execution C. Price improvement D. Prudent man rule

9. A "Best execution" is an investor protection requirement that obligates a broker-dealer to exercise reasonable care to execute a customer's order in a way to obtain the most advantageous terms for the customer.

9. A long stock position is considered __________, and a short stock position is considered __________. A. Bearish, bearish B. Bearish, bullish C. Bullish, bearish D. Bullish, bullish

9. C A long stock position is considered bullish, and a short stock position is considered bearish.

A broker-dealer is permitted to accept payment for a new issue from a new customer when the: A - Registration is effective B - Red herring is delivered C - Preliminary prospectus is amended D - Transaction takes place during the cooling-off period

A

A firm is a participant in a public offering. To sell a substantial amount of the securities to its customers, the firm agrees to repurchase the shares at no less than the original sales price. Such agreements are: A - Prohibited as fraudulent and manipulative B - Permissible if the securities are deposited into escrow C - Prohibited unless the firm immediately sets aside funds for the repurchase D - Permissible if the customers retain the right to sell the securities into the open market

A

Broker-dealer ABC seeks to underwrite a municipal securities offering by a local town in which the major is currently seeking re-election. A municipal finance professional who resides in the town and works for ABC contributed $200 to the mayor's re-election campaign 13 months ago. If the employee wants to support the campaign further without impacting the municipal securities business of the firm, he is permitted to make an additional contribution of up to what amount? A - $50 B - $100 C - $150 D - $250

A

Call protection is most valuable to a bond owner when bond prices are generally: A - Rising B - Falling C - Stable D - Fluctuating

A

In the over-the-counter market, the term "spread" refers to the difference between the: A - Bid and asked prices B - Offered and asked prices C - Indicated and firm prices D - Opening and closing prices

A

In which of the following situations is an employee of a public company permitted to trade upon information without violating insider trading laws? A - An independent securities analyst explains to the employee why the earnings for the company next quarter could be markedly poorer than expected B - The employee's neighbor works for the government and informs him that the government will award a major contract to the company next month C - The employee's spouse works in the corporate headquarters and shares with him a conversation overheard at work regarding possible acquisition targets D - A coworker informs the employee of a conversation he had with the administrative assistant to the company's general counsel regarding key developments in an ongoing class action lawsuit

A

The computation of dollar prices are accrued interest on municipal bonds is normally on what calendar basis? A - 30/360 B - 30/365 C - Actual/360 D - Actual/365

A

Upon expiration, all in-the-money S&P 100 Index call options are settled by the delivery of which of the following? A - Cash B - S&P 100 Index Stocks C - S&P exchange-traded fund (ETF) D 0 The same number of S&P 100 index put options

A

What is the cost basis of an inherited mutual fund? A - The net asset value (NAV) of the shares when the owner dies B - The NAV 30 days after the owner's death C - The same cost basis as the deceased D - The same cost basis as the deceased plus capital gains distribution

A

When a broker-dealer charges a commission on a securities transaction, it has acted as: A - An agent B - A principal C - An underwriter D - A market maker

A

When selling a fixed amount of a base currency to purchase a counter currency, which of the following factors is primarily used to determine how much of the counter currency the customer will receive? A - The sport exchange rate B - The counter currency's inflation rate C - The trade balance between the two countries D - The credit rating of the counter currency's government

A

Which of the following actions is required of a broker-dealer's anti-money laundering (AML) program? A - Designating an AML compliance officer B - Filing of the firm's AML program with FINRA C - Conducting background checks of all employees D - Identifying to regulators the employees responsible for Currency Transaction Reports (CTRs)

A

Which of the following actions is required when a firm receives cash in excess of $10,000 from one customer in one business day? A - File a Currency Transaction Report (CTR) B - File a Suspicious Activity Report (SAR) with the SEC C - Issue a cash transfer receipt to the customer D - Place a five-day hold on the customer's account for the funds received

A

Which of the following considerations should a registered representative explain to a customer when recommending a 529 college savings plan? A - The potential deductibility of contributions from state taxes B - The potential deductibility of contributions from federal taxes C - The income eligibility restrictions to contribute to the account D - The rights of the account beneficiary to the assets at the age of majority

A

The last transaction in XYZ 5.50s 2030 was at 102. This bond is selling at: A - Par B - A premium C - A discount D - Asset value

B

A married couple who earned income that exceeded $300,000 in each of the prior two years and reasonably expects the same for the current year is A - A qualified investor B - An accredited investor C - An institutional investor D - A qualified institutional buyer (QIB)

B

A transaction in which a writer covers a position by purchasing an option is called: A - A closing sale B - A Closing Purchase C - An opening sale D - An opening purchase

B

All of the following risks apply to both foreign and domestic debt instruments except: A - political B - exchange C - repayment D - interest rate

B

An associated person who is not a registered representative is permitted to engage in which of the following activities? A - Accept an unsolicited customer order B - Provide account opening forms to a new customer C - Discuss the attributes of a particular investment product D - Prequalify prospective customers as to investment objectives

B

An investor owns 100 shares of XYZ common stock at the current market price of $50 per share. If XYZ conducts a 1-for-2 reverse stock split, the investor's post-split stock position will be: A - 50 shares at $25 per share B - 50 shares at $100 per share C - 200 shares at $25 per share D - 200 shares at $100 per share

B

Direct participation programs (DPPs) provide: A - Liquidity and transparency B - Exposure to non-correlated assets with steady returns C - Market-related correlations with higher average returns D - Investors with direct purchases of stock from public companies

B

FinCEN accomplishes its mission to safeguard the financial system from the abuses of financial crime, including terrorist financing and money laundering, by enforcing: A - SEC regulations B - the Bank Secrecy Act C - The securities Act of 1933 D - The investment company act of 1940

B

If a market maker posts a quote of 10.00 - 10.10 [25x10], which of the following actions if the market maker willing to take? A - Sell 1,000 shares at $10.00 and buy 2,500 shares at $10.10 B - Buy 2,500 shares at $10.00 and sell 1,000 shares at $10.10 C - Sell 100 shares at $10.00 and buy 250 shares at $10.10 D - Buy 250 shares at $10.00 and sell 100 shares at $10.10

B

No-load mutual funds may have lower expense ratios than load mutual funds for which of the following reasons? A - No-load funds do not charge 12b-1 fees B - No-load funds are not permitted to charge 12b-1 fee greater than 25 basis points C - Sales charges for load mutual funds increase their annual expenses ratios D - Fund management fees for no-load funds are always lower than load funds

B

Regular way settlement on Treasury bonds is: A - Same day B - Next business day C - Second business day (skip day) D - Fifth business day

B

The call provision of a bond stipulates which of the following factors? A - Maturity Date B - Call Date and call price C - Original issue discount D - Coupon rate and call date

B

Which of the following outcomes are possible for the writer of a covered call option? A - Profit limited and loss limited B - Profit limited and loss unlimited C - Profit unlimited and loss limited D - Profit unlimited and loss unlimited

B

Which of the following represents the effect of a stock split? A - The price per share of common stock increases B - The price per share of common stock decreases C - Each stockholder's proportionate ownership decreases D - Each stockholder's proportionate ownership increases

B

Which of the following stakeholders has first claim priority in a Chapter 11 proceeding? A - Equity holders B - Secured debt holders C - Unsecured debt holders D - Administrative claim holders

B

Which of the following statements is true about all U.S. government agency issues? A - They are exempt from federal taxes B - They are exempt from registration under the Securities Act of 1933 C - They are issued in bearer form only D - They are general obligations of the federal government

B

Which of the following statements is true regarding a customer account held as tenants in common? A - The owners must equally share the assets B - The ownership of the decedent's assets is governed by their will or state law C - Tenants in common is permitted for use only for two individuals D - Tenants in common is permitted for use only for married couples or family members

B

Which of the following statements is true regarding the concept of an annuity contract? A - Payouts of an investment in a nonqualified annuity are all income-tax-free B - The kind of annuity selected partly determines the payment amounts to the annuitant C - In the case of life annuity contracts, the age and sex of annuitant do not affect the payment amounts D - The amounts of the periodic payments to the annuitant are determined by the performance of the insurance company's investment returns

B

A member of a stock exchange responsible for providing liquidity in a security by being willing to buy and sell at all times is known as A - A broker B - An underwriter C - A market maker D - A transfer agent

C

A registered representative (RR) wants to participate in a private securities transaction. Which of the following actions must the RR take? A - Request a meeting with his supervisor to lay out the structure of the deal B - Call his supervisor and inform her of the deal and the RR's potential involvement C - Send written notice to his supervisor outlining the activity and compensation structure D - Initiate the paperwork required by the policies of the firm once the activity has begun

C

A registered representative (RR) who wants to place a mutual fund advertisement must obtain written approval from which of the following parties? A - FINRA B - Each state's securities administrator C - A registered principal of the RR's firm D - The investment management division of the appropriate regional SEC office

C

At the time of issuance, which of the following securities normally has the longest period to expiration? A - Rights B - Options C - Warrants D - Repurchase agreements

C

Company ABC announces a 20% stock dividend for its common shareholders. if a customer holds 1,000 shares at $50.00, what is the new price and number of shares following the payment of the stock dividend? A - 800 shares at $62.50 B - 1,000 shares at $41.67 C - 1,200 shares at $41.67 D - 1,200 shares at $50.00

C

In a period of low inflation and economic recession, the Federal Reserve is expected to take which of the following action? A - Decrease taxes B - Raise the federal funds rate C - Buy bonds in the open market D - Require banks to increase reserves

C

The owner of which of the following products is most exposed to inflationary risk? A - Utility stocks B - Treasury bills C - Treasury bonds D - Blue chip industrials

C

The redemption value of an open-end investment company's shares is based on the: A - Previous offering price B - Previous closing net asset value (NAV) C - NAV computed after the order is received D - Offering price computed after the order is received

C

Under FINRA rules, which of the following activites by a registered representative requires written notification to her firm? A - Volunteering as a youth soccer league coach B - Gambling resulting in winnings in excess of $10,000 C - Driving part time for a transportation network company on weekend D - Selling non proprietary investment company securities products through her firm

C

Under Rule 144A, an issuer of restricted stock is permitted to sell to which of the following investors? A - Financial institutions B - Accredited investors C - Qualified institutional buyers (QIBs) D - Nonaccredited investors who have previously purchased restricted stock

C

Under a system of statutory voting, a common stockholder has as many votes for each vacancy on the board of directions as the number of: A - Positions vacant on the board B - Directors present at the meeting C - Shares owned by the stockholder D - Proxies available for voting by the board

C

Under industry regulations, an initial purchase of $3,000 of common stock in a margin account requires a cash deposit of: A - $750 B - $1,500 C - $2,000 D - $3,000

C

Under normal circumstances, a customer's letter of intent on a mutual fund purchase is valid for what maximum period of time? A - 3 months B - 6 months C - 13 months D - 24 months

C

Under the securities Act of 1933, registration is required for which of the following securities? A - Eurodollar bonds B - Municipal securities C - American Depository Receipts (ADRs) D - Securities issued by the federal government

C

Which of the following investments are generally traded according to their average life rather than their stated maturity dates? A - Corporate bonds B - Government bonds C - Asset-backed securities D - Fixed-rate capital securities

C

Which of the following security types provides investors with a state maturity date, a floating interest rate, and an option to put the security back to a financial intermediary on a daily or weekly basis? A - Equity put option B - Perpetual preferred stock C - Variable rate demand note D - Tax-deferred variable annuity

C

Which of the following statements is true about treasury stock? A - It has voting rights B - It receives dividends C - It is issued stock that has been subsequentl reacquired by the corporation D - it is authorized stock that has not been issued and is held in the corporation's treasury

C

A company announces a tender offer to its shareholders with the intent to buy a maximum of 1 million shares of its outstanding stock at $10 per share and sets no minimum number of shares to be purchased. An investor wants to participate in this offer and tenders his 1,000 share position, At the close of the offer period, only 900,000 shares have been tendered. How many, if any, of the investor's shares will the company purchase? A - 0 B - 100 C - 900 D - 1,000

D

A customer buys 1 ABC Jan 35 put for a premium of $3 and simultaneously buys 100 shares of ABC stok for $35 per share. The customer will break even when the stock is selling at what price per share at expiration? A - $3 B - $32 C - $35 D - $38

D

A registered representative is reviewing the following portfolio: -30% ABC Energy Company -30% XYZ Health Care Company -10% Money market funds Which of the following risks is inherent in this portfolio? A - Credit B - Liquidity C - Political D - Nonsystematic

D

At issue, which of the following debt securities mature in one year or less? A - FNMA bonds B - GNMA bonds C - Treasury notes D - Money market instruments

D

Blue-sky laws are regulated by which of the following entities? A - SEC B - MSRB C - FINRA D - State securities regulators

D

In the event of a stock split, which of the following parties is required to maintain a record of the shareholders eligible to receive the additional shares? A - DTC B - Issuer C - Custodian D - Transfer agent

D

Roth 401(k) and Roth individual retirement account (IRA) plans share which of the following features? A - Contributions are made pretax B - Neither account has maximum contribution limits C - Neither account is subject to early distribution penalties D - Qualified distributions are excluded from federal income tax

D

Stability in the value of a debt portfolio is greatest when: A - Interest rates are rising B - Interests rates are falling C - Maturities of the debt securities are long D - Maturities of the debt securities are short

D

Under FINRA rules, noncash compensation connected with the sale of variable contracts includes all of the following items except: A - gifts B - meals C - lodging D - commissions

D

Under Federal Reserve Regulation T, a 90-day restriction is imposed when a customer opens and closes which of the following positions? A - The same stock on the same day in a margin account B - The same stock in different accounts on the same day C - The same stock in different accounts in the same household D - The same stock in a cash account without paying for the stock in full

D

Under SEC Regulation S-P (Consumer Privacy), which of the following information must a firm include in its customer privacy and opt-out notices? A - The address of the firm's website B - The fee to opt out from the privacy program C - The deadline by which to opt out from the privacy program D - The policies to protect the security of nonpublic information

D

Which of the following Form U4 reportable events results in a statutory disqualification? A - A personal bankruptcy filing B - A written customer complaint alleging misappropriation of assets C - A felony theft charge D - A felony conviction for a driving under the influence (DUI) charge

D

Which of the following actions violates FINRA rules regarding selling away? A - Recommending to a customer the services of another broker-dealer (BD) B - Sharing commissions with another registered representative in the same BD C - Advising customers to buy mutual funds at amounts just below a discount level D - Engaging in private securities transactions without written prior consent of the employing BD

D

Which of the following communications with the public is considered misleading? A - Historical illustrations based on factual performance B - Product comparisons that illustrate material differences C - A research report that includes a buy recommendation for a specific security D - Literature providing 10-year performance information to support predictions of future performance

D

Which of the following comparisons best described the essential difference between a primary distribution and a secondary distribution? A - A primary distribution can only be made for equity issues while a secondary distribution can be made for both equity and debt issues B - A primary distribution can only be made for issues that exceed a specified dollar amount while a secondary distribution can be made for issues of all sizes C - A primary distribution must be made at a fixed public offering price (POP) while a secondary distribution must be made at the current market price D - A primary distribution involves a sale of newly issued shares by the issuer while a secondary distribution involves the sale of already issued and outstanding shares

D

Which of the following products is adversely impacted if the issuer's credit rating is downgraded A - Mutual funds B - Unit investment trusts (UITs) C - Exchange-traded Funds (ETFs) D - Exchange-traded notes (ETNs)

D

Which of the following responses describes an advantage of index funds? A - The protection of principal during bearish markets B - High turnover to take advantage of market opportunities C - Higher management fees due to active management of the fund D - Lower management fees due to passive management of the fund

D

Your customer wants to invest in a conservative income‐producing investment and is inquiring about GNMAs. She wants to know the minimum dollar amount required to purchase a pass‐through certificate. You should tell her: $1,000. $10,000. There is no minimum; you can invest almost any sum. $5,000.

(a) The minimum dollar amount to purchase a GNMA pass‐through certificate is $1,000.

In response to a customer's request for information on how inflation will affect their return realized from their semiannual coupon payments, you would look at the: Real interest rate. Adjusted interest rate. Interest conversion rate. Current interest rate.

(a) The real interest rate will determine the return after inflation.

The SEC has been reviewing a company's registration statement and would like clarification on a few items. The SEC would most likely: Call the company. Issue a stop order. Issue a deficiency letter. Call the lead underwriter

(c) Anytime the SEC wants more information, it would most likely issue a deficiency letter.

Which of the following are bearish? Call seller. Put seller. Call buyer. Put buyer. II and III. II and IV. I and IV. I and II.

(c) Call sellers and put buyers are both bearish. They want the value of the stock to fall.

During an inflationary period, the price of which one of the following will fall the most? Preferred stock. Treasury bills. Treasury bonds. Common stock.

(c) During an inflationary period, the price of a Treasury bond will fall the most. The fixed‐income security with the longest maturity will change the most in price as interest rates change.

All of the following indicate a downturn in the business cycle, except: Rising inventories. High consumer debt. Falling inventories. Falling stock prices.

(c) Falling inventories are a sign of a pickup in the economy.

Your brokerage firm acts as a market maker for several high‐volume stocks that are quoted on the Nasdaq. What is the firm's consideration for being a market maker? Commission. Fees. Spread. 5%

(c) Firms that act as market makers in Nasdaq securities are trying to make the spread, which is the difference between the bid and the ask

An investor buys 100 shares of XYZ 7% convertible preferred stock which are convertible into XYZ common stock at $20 per share. How many shares of common stock are there upon conversion? 5. 400. 500. 5,000.

(c) First you must determine the number of shares. Par / conversion price = 100 / 20 = 5, multiplied by the number of preferred shares: 5 × 100 = 500.

An investor wires $10,000 into his mutual fund on Tuesday, March 11, and the money is credited to his account at 3 p.m. He will be the owner of record on: Friday, March 14. Wednesday, March 12. Tuesday, March 11. Tuesday, March 18.

(c) New shares will be created for the investor as soon as the mutual fund company receives the money. The investor becomes an owner of record on that day.

Which one of the following debt securities pays interest? Commercial paper. T‐bill. Industrial revenue bond. Banker's acceptance.

(c) Of all the choices listed, only an industrial revenue bond pays interest; all of the other choices are issued at a discount.

The Federal Reserve Board sets all of the following except: Monetary policy. Reserve requirement. Governmental spending. Discount rate.

(c) The Federal Reserve sets all of those except government spending.

An investor purchased a Treasury bond at 95.03. How much did he pay for the bond? $ 9,530.00. $ 9,500.9375. $ 950.9375. $ 953.00.

(c) The investor purchased the Treasury bond at 95.03 or 95‐3/32% of $1,000 = $950.9375

An investor who buys a 7% cumulative preferred stock will receive semiannual dividends of: A - $7 per share. B - 7% of the corporate profits. C - $3.50 per share. D - 3.5% of the corporate profits.

(c) The investor who buys a 7% preferred stock is entitled to $7 per year or $3.50 every six months.

A firm participating in the offering of a private placement may sell the private placement to no more than _________________ nonaccredited investors in any 12‐month period? 12. 6. 35. 15.

(c) The number of nonaccredited investors is limited to 35 in any 12‐month period.

XYZ has 8% subordinated debentures trading in the market place at $120. They are convertible into XYZ common stock at $25 per share. What is the parity price of the common stock? 29. 31. 30. 28.

(c) The parity price is found by determining the number of shares that can be received upon conversion par / conversion price = 1,000 / 25 = 40 shares. Then the parity price equals the current market value of the convertible / no. of shares, 1,200 / 40 = $30.

Which of the following is NOT true of money market instruments? They are highly liquid fixed‐income securities. They are issued by corporations with high credit ratings, and are thus considered safe. They are considered risky because of short‐term maturities. They are a method used to obtain short‐term financing.

(c) The short‐term maturity and the fact that the issuers have solid credit ratings make money market instruments very safe.

The government has two tools it can use to try to influence the direction of the economy. They are: Monetary policy and fiscal policy. Prime rate policy and fiscal policy. Monetary policy and prime rate policy. Fiscal policy and money market policy.

(a) The two tools of the government are monetary policy, which is con‐ trolled by the Federal Reserve Board and controls the money supply, and fiscal policy, which is determined by the president and Congress and controls government spending and taxation.

When making markets over the counter, the firm is acting in what capacity? Dealer. Both. Neither. Broker.

(a) When acting as a market maker, the firm is trading for its own account and is acting as a dealer.

Once a company decides to raise long‐term capital to meet its needs, it will do which of the following? Approach the money market to determine how much capital can be raised. Hire an underwriter to advise the issuer about the type of securities to issue. Hire a dealer to issue stock for public purchase. Hire a broker to issue stock for public purchase.

(b) A business must first hire an underwriter to advise the issuer about the types of securities to issue.

A fixed annuity guarantees all of the following except: Income for life. Protection from inflation. Rate of return. Protection from investment risk.

(b) A fixed annuity does not provide protection from inflation. If inflation rises, the holder of a fixed annuity may end up worse off due to the loss of value of the dollar

The type of bond that is secured by real estate is called a: Real estate trust certificate. Mortgage bond. Equipment trust certificate. Collateral trust certificate.

(b) A mortgage bond is secured by real estate.

All of the following are rights of common stockholders, except: A - Right to elect the board of directors. B - Right to vote for executive compensation. C - Right to vote for a stock split. D - Right to maintain their percentage of ownership in the company.

(b) A stockholder does not get to vote directly for executive compensation.

A syndicate may enter a stabilizing bid: Whenever the price begins to decline. At or below the offering price. To ensure an increase from the offering price. To cover overallotments only.

(b) A syndicate may only enter a stabilizing bid at or below the offering price.

You are long 10,000 shares of XYZ at 42 and are concerned about a market decline; you would like to take in some additional income. You should: Sell 10 XYZ Oct 45 puts. Sell 100 XYZ Oct 45 calls. Sell 100 XYZ Oct 45 puts. Sell 10 XYZ Oct 45 calls.

(b) To gain some protection and take in premium income, you would sell 100 XYZ Oct 45 calls.

Which of the following is NOT a corporate money market instrument? Negotiable certificates of deposit. Treasury bills. Federal funds. Commercial paper.

(b) Treasury bills are government money market instruments, not corporate money market instruments

If a 5% stock dividend is paid to an investor who owns 800 shares of stock already, the investor will receive how many shares? 4 shares. 8 shares. 40 shares. 80 shares.

(c) 800 shares × 5% = 40 shares.

A Regulation A offering as amended by the Jobs Act pertains to an: Intrastate offering of securities. Offering of bonds. Offering of $50,000,000 or less. Offering of $3,000,000 or less

(c) A Regulation A offering covers an offering of $50,000,000 or less in a 12‐month period.

During an underwriting of a hot issue, the syndicate exercises its greenshoe provision. This will allow the syndicate to buy what additional percentage of the offering? 20%. 25%. 15%. 10%.

(c) A greenshoe provision allows the syndicate to purchase up to an additional 15% of the offering from the issuer.

As the owner of a cumulative preferred stock, an investor would have all of the following rights, except: A - Voting if dividends are missed for a significant period of time. B - The right to receive past dividends not paid by the corporation. C - The right to exchange the preferred for the underlying common shares. D - The right to receive the past dividends before common holders receive a dividend.

(c) A holder of a cumulative preferred has all the rights listed, except the right to convert the preferred into common stock.

A mutual fund has been seeking to attract new customers to invest in its growth fund. They have been running an advertising campaign that markets them as a diversified mutual fund. How much of any one company may they own? 15%. 5%. 10%. 9%.

(c) A mutual fund calling itself a diversified fund is limited to owning no more than 10% of any one company.

A mutual fund's custodian bank does which of the following? Holds customers' securities. Cancels certificates. Maintains records for accumulation plans. Issues certificates.

(c) A mutual fund's custodian maintains books and records for accumulation plans.

INTC has been hitting a lot of resistance at $30. A technical analyst who wants to buy the stock would most likely place what type of order? Limit order to buy at $30. Market order. Buy stop at $31. Buy limit at $29.

(c) A technical analyst would want to buy the stock when it breaks through resistance.

All of the following are reasons a corporation would attach a warrant to their bond, except to: Save money. Make the bond more attractive. Increase the number of shares outstanding when the warrants are exercised. Lower the coupon.

(c) All of the choices listed are reasons a corporation would attach warrants to their bonds, except to increase the number of shares outstanding.

For an insider to sell unregistered stock under an exemption from registration with the SEC, Form 144, Notice of Offering, which contains certain information, must be filed with the SEC. The insider can sell securities during the period of time in which the notice of offering is effective, which is: 60 days. 6 months. 90 days. 12 months.

(c) An insider may sell securities under Rule 144 for 90 days.

An investor with $20,000 invested in the XYZ growth fund is: A stockholder in XYZ. An owner of XYZ. An owner of an undivided interest in the XYZ growth portfolio. Both an owner of XYZ and an owner of an undivided interest in the XYZ growth portfolio.

(c) An investor in a mutual fund portfolio has an undivided interest in that portfolio and is not an investor or stockholder in the fund company itself.

An investor holding an 8% subordinated debenture will receive how much at maturity? $1,000. $1,080. $1,040. Depends on the purchase price.

(c) An investor who has purchased an 8% corporate bond will receive the principal payment plus the last semiannual interest payment at maturity for a total of $1,040.

A bullish investor would enter which of the following orders? A sell limit thinking that the stock price will rise. A sell stop below the market. A buy stop above the market. DNR GTC.

(c) An investor who is bullish would most likely enter a buy stop above the market.

ABC common stock declined dramatically in value over the last quarter but the dividend it declared for payment this quarter has remained the same. The dividend yield on the stock has: A - Not changed because the board has to declare the dividend amount. B - Gone down because the yield is a stated rate. C - Gone up as the price of ABC has fallen. D - Been fixed at the time of issuance.

(c) The yield on the stock will have gone up as the price has fallen because the dividend has remained constant.

A no‐load mutual fund may charge a 12B‐1 fee that is: Up to .25 of 1% of the NAV. Less than .25 of 1% of the NAV. Up to .25 of 1% of the POP. Less than .25 of 1% of the POP.

(a) A 12B‐1 fee may be up to ¼ of 1% of the NAV

A bank with a shortfall meeting their reserve requirement could borrow money from another bank and pay the: Federal funds rate. Broker call loan rate. Prime rate. Discount rate.

(a) A bank may borrow money from another bank to meet their reserve requirement and consequently pay the other bank the federal funds rate.

Corporations may do all of the following, except: Issue preferred stock only. Issue nonvoting common stock. Sell stock out of the treasury. Repurchase its own shares.

(a) A corporation must issue common stock before it issues any preferred stock.

A decline in the GDP must last at least how long to be considered a recession? Two quarters. One quarter. Six quarters. Four quarters.

(a) A decline in the gross domestic product must last at least two quarters or 6 months to be considered a recession.

A long‐term growth fund has a portfolio turnover ratio of 25%. How often does the fund replace its total holdings? Every 4 years. Once a year. Every 4 months. Every 4 months.

(a) A fund with a portfolio turnover ratio of 25% replaces its portfolio every 4 years.

A red herring given to a client during the cooling‐off period will contain all of the following, except: Proceeds to the company. Use of proceeds. Biographies of officers and directors. A notice that all the information is subject to change.

(a) All of the answers listed will appear in the preliminary prospectus, except the offering price and the proceeds to the company.

Which of the following is NOT a type of offering? Rule 149 offering. Subsequent primary offering. Secondary offering. Combined offering.

(a) All of the choices listed are types of offerings except for Rule 149.

An investor would expect to realize the largest capital gain by buying bonds that are: Long‐term when rates are high. Short‐term when rates are low. Short‐term when rates are high. Long‐term when rates are low.

(a) An investor would expect to realize the largest gain by purchasing bonds when rates are high. The bond with the longest time left to maturity will become worth the most as interest rates fall.

When is the interest on an EE savings bond paid? When redeemed. Annually. Quarterly. Monthly.

(a) EE savings bonds are sold at a discount and at maturity are redeemed at face value, which includes the interest income.

It may be necessary for a company to repurchase some of its stock, to increase its treasury stock, for which one of the following reasons: A - To maintain control of the company. B - To allow the company to pay out smaller dividends. C - To increase the funding in the company's treasury. D - To reassure its investors that all is well.

(a) In addition to maintaining control, a company may want to increase its earnings per share, fund employee stock option plans, or use shares to pay for a merger or acquisition.

An investor has purchased 10 corporate bonds at a price of 135. At the end of the day, the bonds are quoted at 136.25. How much have the bonds risen in dollars? $125. $12.50. $1.25. $.125.

(a) One bond point is worth $10.00; 1.25 points, therefore, is worth: $12.50 × 10 bonds = $125.

Which of the following is NOT a type of underwriting commitment? Primary commitment. Standby commitment. Best efforts commitment. Firm commitment.

(a) Primary commitment is not a type of underwriting commitment.

Which of the following subjects the investor to unlimited risk? Selling stock short. Converting a bond into the underlying common stock. Buying a speculative bond. Selling common stock long.

(a) Selling stock short will expose an investor to unlimited risk because there is no limit as to how high a stock price can go.

Which of the following is true of the DMMs on the NYSE? They work for themselves. They are appointed by a vote of the company's board. They work for the exchange. They work for the company whose stock they trade.

(a) Specialists on the NYSE work for themselves or for a specialist firm.

The OCC is: Options Clearing Corporation. Options Counseling Committee. Options and Claims Corporation. Options Clearing Committee.

(a) The Options Clearing Corporation issues all option contracts and guarantees their performance.

An investor buys $10,000 of 10% corporate bonds with 5 years left to maturity. The investor pays 120 for the bonds. What is the investor's current yield? 8.33%. 11.1%. 6%. 9.2%.

(a) The current yield is found by dividing the annual income by the current market price. In this case $100 / $1,200 = 8.33%

The inside market is: Highest offer. Lowest offer. Highest bid. Lowest bid. II and III. I and II. I and IV. I and III.

(a) The inside market is the highest bid and the lowest offer.

An investor has purchased shares of a foreign company through an ADR. Which of the following is not true? The ADR may represent one or more shares of the company's common stock. The dividend will be paid in U.S. dollars. The investor may elect to exchange the ADR for the underlying common shares. The investor is subject to currency risk.

(b) An ADR may represent more than one share of the company's common stock and may be exchanged for the ordinary common shares. The dividend, however, is paid in the foreign currency and is received by the investor in U.S. dollars; as a result, the investor is subject to currency risk.

Authorized stock is all of the following, except: A - It is the maximum number of shares a company may sell. B - It is arbitrarily determined at the time of incorporation and may not be changed. C - It may be sold in total or in part when the company goes public. D - It may be sold to investors to raise operating capital for the company.

(b) Authorized stock is all of the answers listed, except the number of authorized shares may be changed by a vote of the shareholders.

Which of the following is not true regarding American Depositary Receipts (ADRs)? They are receipts of ownership of foreign shares being held abroad in a U.S. bank. Each ADR represents 100 shares of foreign stock, and the ADR holder may request delivery of the foreign shares. ADR holders have the right to vote and to receive dividends that the foreign corporation declares for shareholders. The foreign country may issue restrictions on the foreign ownership of stock.

(b) Each ADR represents between one to 10 shares, and ADR holders have the right to vote and receive dividends. Foreign governments put restrictions on the foreign ownership of stock from time to time.

The city of Chicago is seeking to raise $100 million through the sale of general obligation bonds and is seeking an underwriter for the issue. Which of the following is correct? The issue will be awarded through a negotiation. The issue must be advertised and provide terms for bidding. The issue will be underwritten on a best efforts basis if the city's bonds are not in high demand. The city is looking to the lower true interest cost to finance the issue.

(b) General obligation bonds will be advertised in the daily Bond Buyer and the official notice of sale will provide details on the bidding procedure.

All of the following are true regarding the Federal National Mortgage Association (Fannie Mae) except: It purchases mortgages and packages them to create mortgage‐backed securities that pay interest semiannually. It provides an investment free of federal, state, and local taxes. It is a public for‐profit corporation. Its purpose is to earn a profit by providing mortgage capital.

(b) Interest earned by investors on FNMA securities is taxable at all levels: federal, state, and local.

An investor buys 10 XYZ Nov 75 calls at 4:10 on Monday, May 11. The trade will settle on: Thursday, May 14. Tuesday, May 12. Monday, May 11. Monday, May 18.

(b) Option trades settle the next day.

XYZ has just gone public and is quoted on the Nasdaq Capital Market securities market. Any investor who buys XYZ must get a prospectus for how long? 30 days. 25 days. 60 days. 45 days.

(b) Purchasers of stock that has just gone public must get a prospectus for 25 days if the stock is Nasdaq listed.

The state of Massachusetts is seeking to raise $300 million through the sale of revenue bonds to repair the roadways. Which of the following is correct? These bonds will be subject to a statutory debt limit. These bonds will be issued through a negotiation. The offering will be advertised in the daily bond buyer. The underwriters will be required to submit sealed bids.

(b) Revenue bonds are issued through a negotiated underwriting process. The issuer will select the underwriter they would like to have offer the bonds and negotiate the terms with the underwriter directly.

Rule 145 covers which of the following? Stock splits. Stock swaps. Reverse splits. Changes in par value.

(b) Rule 145 covers mergers involving a stock swap or offer of another company's securities in exchange for its current stock.

Which of the following issues standardized options? Exchanges. OCC. Company. Nasdaq.

(b) The OCC (Options Clearing Corporation) issues all standardized options.

An investor buys a 10% preferred stock at 110. What is its current yield? A - 10.4%. B - 9.1%. C - 10%. D - 9.5%.

(b) The current yield is found by using the following formula: annual income / current market price of $10 / $110 = 9.1%.

Your customer is long 100 shares of MSFT. The investor wants to protect the position without spending any additional money, what should he do? Buy a call. Sell a call. Sell a put. Buy a put.

(b) The investor is long the stock and wants to protect his position. The key to the question is that he does not want to spend any additional money. In this case he must sell a call. He will receive partial protection in the amount of the premium received.

Economic theories believe all of the following to be true, except: As supply rises, prices tend to fall. As supply rises, prices tend to rise. A moderately increasing money supply promotes price stability. As demand rises, prices tend to rise.

(b) The main theory of economics is one of supply and demand; if the supply outpaces the demand, the price of the goods will fall.

The money market is a place where issuers go to: Obtain long‐term financing. Obtain short‐term financing. Offer higher interest rates for a higher yield. Exchange money market instruments to their mutual benefit.

(b) The money market is a place where issuers go solely for short‐term financing, typically under a year.

ABC Technologies, a very volatile stock, closes at $180 per share. Your customer has placed an order to sell 500 ABC at 165 stop limit 160 GTC. After the close, the company announces bad earnings and the stock opens at 145. What happened to your customer's order? It has been canceled because the stock price is below the limit price. It has been elected and has become a limit order. It has been elected and executed. It has been canceled because the stock price is below the stop price.

(b) The order has been elected because the stock has traded through the stop price. The order has now become a limit order to sell the stock at 160.

An ABC corporate bond is quoted at 110 and is convertible into ABC common at 20 per share parity. Price for the stock is: 21. 22. 23. 24.

(b) The parity price of the stock is found by using the following formulas: no. of shares = PAR / CVP, 1,000 / 20 = 50, parity price = CMV of bond / no. of shares = 1,100 / 50 = 22.

The city of Miami is seeking to raise $10 million through the sale of general obligation bonds to repair the high school's football field. The bonds are going to be issued ex‐legal. Which of the following is correct? The bonds received an unqualified legal opinion. The bonds received a qualified legal opinion. The bonds received no legal opinion. The bonds' legal claim to the tax revenue is in doubt.

(c) This is a very small bond issue. As such it would not make sense in many cases to pay substantial legal fees to obtain a legal opinion. These bonds are said to be ex‐legal because no legal opinion was ever obtained.

Your customer buys a U.S. T‐bond at 103.16. How much did he pay for the bond? $1,031.60. $103.16. $1,035.00. $10,316.00.

(c) T‐bonds are quoted as a percentage of par to 32nds of 1%. A quote of 103.16 = 103

A company doing a preemptive rights offering would most likely use what type of underwriting agreement? Best efforts. Firm commitment. All or none. Standby.

(d) A company doing a rights offering will use a standby underwriting agreement whereby the underwriter will "stand by" ready to purchase any shares not purchased by shareholders.

Which of the following could trade in the money market? Short‐term equity. Newly issued corporate bonds. Newly issued options contracts. A Treasury note issued nine years ago.

(d) A high‐quality debt instrument with less than one year to maturity, regardless of its original maturity, may trade in the money market.

A corporation may pay a dividend in which of the following ways? Choose the most complete response. Stock. Cash. Stock of another company. All of the above.

(d) All choices listed are ways that a company can pay a dividend.

A syndicate has published a tombstone ad prior to the issue becoming effective. Which of the following must appear in the tombstone? A statement that the registration has not yet become effective. A statement that the ad is not an offer to sell the securities. Contact information. No commitment statement. III and IV. II and III. I and II. I, II, III, and IV.

(d) All of the items listed must appear in the tombstone ad.

During a new issue registration, false information is included in the prospectus to buyers. Which of the following may be held liable to investors? Officers of the issuer. Accountants. Syndicate members. People who signed the registration statement. I and III. I, II, III, and IV. I, II, and III. I, II, III, and IV.

(d) All of the parties listed may be held liable to the purchasers of the new issue.

All qualified dividends for ordinary income earners are: Taxed as ordinary income each year. Tax‐free income. Taxed as special interest‐free income. Taxed at a set rate of 15%.

(d) All qualified dividends received by ordinary income earners are taxed at a rate of 15% for the year in which they were received.

Which of the following may NOT trade on the floor of the NYSE? Two‐dollar broker. Regular member. Commission house broker. Allied member.

(d) An allied member may not trade on the floor. He is only allowed to call himself a member and have electronic access to the exchange.

A doctor makes the maximum contribution to his Keogh plan while earning $300,000 per year. How much can he contribute to an IRA? $55,000. $17,000. $9,000. $5,500

(d) An investor may always make a contribution to his IRA as long as he has earned income

A company you own common stock in has just filed for bankruptcy. As a shareholder, you will have the right to receive: The par value of the common shares. New common shares in the reorganized company. A percentage of your original investment. Your proportional percentage of residual assets.

(d) As a common stockholder, you will have the right to receive your percentage of any residual assets.

Which bonds are issued as a physical certificate without the owner's name on them and require whoever possesses these bonds to clip the coupons to receive their interest payments as well as surrender the bond at maturity in order to receive the principal payment? Registered bonds. Book entry/journal entry bonds. Principal‐only registered bonds. Bearer bonds.

(d) Bearer bonds are issued without a name on them, meaning that whoever has possession of the bond may clip the coupons and claim the interest.

Which type of bonds require the investor to deposit coupons to receive their interest payments but have the owner's name recorded on the books of the issuer? Registered bonds. Bearer bonds. Book entry/journal entry bonds. Principal‐only bonds.

(d) Bonds registered as to principal only will still require the investor to clip coupons.

An investor is long 1000 shares of OnNet.com at $30 per share. To gain the maximum protection he should: Sell 10 OnNet June 30 calls. Sell 10 OnNet June 30 puts. Buy 10 OnNet June 30 calls. Buy 10 OnNet June 30 puts.

(d) Buying puts will give the investor the most protection from a fall in the stock price. The investor will have set the minimum sales price for the stock as the strike price of the put.

Collateral trust certificates use which of the following as collateral? Real estate. Mortgage. Stocks and bonds issued by the same company. Stocks and bonds issued by another company.

(d) Collateral trust certificates have pledged securities, which they own, issued by another company as collateral for the issue.

Common stockholders do not have the right to vote on which of the following issues? A - Election of the board of directors. B - Stock splits. C - Issuance of additional common shares. D - Bankruptcy.

(d) Common stockholders do not have voting power in the matter of bankruptcy.

Common dividends are all of the following except: A portion of the earnings of the company. A source of income for the investor. Generally paid quarterly. A figure determined by subtracting the current yield from the current market price.

(d) Dividend yield (or current yield) is found by dividing the annual income by the current market price.

Fiscal policy is controlled by: President. FOMC. Congress. FRB. I and IV. I and II. II and IV. I and III

(d) Fiscal policy is controlled by the president and Congress.

Which of the following are true about an option? It is a contract between two parties that determines the time and place at which a security may be bought or sold. The two parties are known as the buyer and the seller. The money paid by the buyer of the option is known as the option's premium. The buyer has bought the right to buy or sell the security depending on the type of option. The seller has an obligation to perform under the contract, possibly to buy or sell the stock depending on the option involved. I, III, and IV. I, II, III, and IV. I, II, and III. II, III, and IV.

(d) I is incorrect in that an option is a contract between two parties, which determines the time and price at which a security may be bought or sold.

Which of the following is NOT a type of order? All or none. Fill or kill. Mini/maxi. Best efforts. I and II. II and IV. I and IV. III and IV.

(d) Mini/maxi and best efforts are types of underwriting commitments, not types of orders

Which of the following may always trade in the money market? T‐bond. T‐note. ADR. Bankers' acceptance.

(d) Only bankers' acceptances have an original maturity of less than a year.

All of the following are bullish for the stock market, except: Falling taxes. Increasing government spending. Increasing money supply. Increasing interest rates.

(d) Rising interest rates are bearish for the stock market.

The state of Texas is seeking to raise $500 million through the sale of general obligation bonds. Which of the following will support the repayment of the bond issue? Property taxes. Ad valorem taxes. User fees. Sales taxes.

(d) The debt service for general obligation bonds issued by the state is supported by revenue received at the state level. This revenue includes sales taxes and income taxes.

Which one of the following interest rates is controlled by the Federal Reserve Board? Prime rate. Federal funds rate. Broker call loan rate. Discount rate.

(d) The discount rate is the rate that is actually controlled by the Federal Reserve Board. All of the other rates are adjusted in the marketplace by the lenders as a result of a change in the discount rate.

The ex‐dividend date on a closed‐end mutual fund is set by the: Board of directors. SEC. Board of governors. FINRA/NYSE.

(d) The ex date is set by the NYSE/FINRA for a closed‐end fund just like for a stock

An investor owns 100 shares of XYZ 8% participating preferred stock. XYZ's common stock pays a quarterly dividend of $.25. How much will the investor earn each year in dividends? A - $825. B - $90. C - $180. D - $900.

(d) The investor will receive $8 per share × 100 shares: $800 plus $1 per share because it is participating, so $800 + $100 = $900.

The maximum amount that a couple may contribute to their IRAs at any one time is: 100% of the annual contribution limit. 200% of the annual contribution limit. 300% of the annual contribution limit. 400% of the annual contribution limit.

(d) The maximum amount that a couple may contribute to their IRAs at any one time is $22,000. Between January 1 and April 15, a contribution may be made for the prior year, the current year, or both: $5,500 × 2 × 2 = $22,000.

The maximum duration for a piece of commercial paper is: 45 days. 10 years. 1 year. 9 months.

(d) The maximum duration for a piece of commercial paper is 9 months, or 270 days.

A corporation in your state wants to sell 1,000,000 shares of stock at $5 per share to investors. Which of the following is NOT true under Rule 147? 80% of corporate assets must be located in the state. 80% of proceeds must be used in the state. 80% of the income must be derived from activity within the state. 80% of the purchasers must be in the state.

(d) Under Rule 147, 100% of the purchasers must be in the state. The issuer must meet one of the doing‐business standards as listed in the other choices.


संबंधित स्टडी सेट्स

Capitalization 3-Which word is incorrectly capitalized?

View Set

Ch. 23: Health Assessment of Children

View Set

PSYC TEST 3: launchpad questions: module 36, 37, 38, 39

View Set

Chapter 16 The Civil War; Section 1 The War Begins, ( From Textbook ; U.S History, Begginings to 1914")

View Set

labce game mode questions 1/6/23 -3

View Set

Chapter 53 Male Reproductive Disorders Prep U

View Set

Sociálna psychológia - 2. Socializácia a 3. Kultúra a spoločnosť

View Set

Chapter 6 Exam - Markets and Social Security

View Set